continuity of monotonically increasing functionf left continuous & strictly increasing; B Borel $implies$ f(B) Borel (or at least Lebesgue Measurable)?Discontinuities of a monotonic function (Baby Rudin)If two functions are both increasing and equal almost everywhere, do they have the same discontinuous points?Doubt in discontinuities of a monotonic function.Bivariate function monotone in each variable $Rightarrow$ continuous a.e.?Characteristics of a monotonic function on a closed interval$ f : D rightarrow mathbbR $ monotone increasing. Cardinality of discontinuity points?Inverse of function, monotone on interval is continuousthe continuity of monotone additive function on $mathbbR$Monotone Functions and Continuities

The Two and the One

Is it unprofessional to ask if a job posting on GlassDoor is real?

Mathematical cryptic clues

What typically incentivizes a professor to change jobs to a lower ranking university?

Is it legal for company to use my work email to pretend I still work there?

Is it tax fraud for an individual to declare non-taxable revenue as taxable income? (US tax laws)

"to be prejudice towards/against someone" vs "to be prejudiced against/towards someone"

Can an x86 CPU running in real mode be considered to be basically an 8086 CPU?

Smoothness of finite-dimensional functional calculus

Why Is Death Allowed In the Matrix?

How does one intimidate enemies without having the capacity for violence?

Prove that NP is closed under karp reduction?

Why can't I see bouncing of a switch on an oscilloscope?

What is the word for reserving something for yourself before others do?

What do you call a Matrix-like slowdown and camera movement effect?

A newer friend of my brother's gave him a load of baseball cards that are supposedly extremely valuable. Is this a scam?

How do I create uniquely male characters?

Why do I get two different answers for this counting problem?

Why doesn't H₄O²⁺ exist?

Approximately how much travel time was saved by the opening of the Suez Canal in 1869?

Theorems that impeded progress

Is it important to consider tone, melody, and musical form while writing a song?

LaTeX closing $ signs makes cursor jump

Which models of the Boeing 737 are still in production?



continuity of monotonically increasing function


f left continuous & strictly increasing; B Borel $implies$ f(B) Borel (or at least Lebesgue Measurable)?Discontinuities of a monotonic function (Baby Rudin)If two functions are both increasing and equal almost everywhere, do they have the same discontinuous points?Doubt in discontinuities of a monotonic function.Bivariate function monotone in each variable $Rightarrow$ continuous a.e.?Characteristics of a monotonic function on a closed interval$ f : D rightarrow mathbbR $ monotone increasing. Cardinality of discontinuity points?Inverse of function, monotone on interval is continuousthe continuity of monotone additive function on $mathbbR$Monotone Functions and Continuities













1












$begingroup$


Let $f:[0,1]to[0,1]$ be monotonically increasing. Which of the following statements is/are true?



  1. $f$ must be continuous at all but finitely many points in $[0,1]$

  2. $f$ must be continuous at all but countably many points in $[0,1]$

  3. $f$ must be Riemann integrable

  4. $f$ must be Lebesgue integrable

I know that set of discontinuities of monotonic function is at most countable so 1 is not true.



If the set of discontinuities is at most countable then how to conclude for the second option that is set of continuities of monotone function. Here domain is [0,1] .So the set of continuities can be uncountable?



Monotone function on $[a,b]$ is Riemann integrable and hence Lebesgue integrable hence option 3 and 4 are correct.










share|cite|improve this question











$endgroup$











  • $begingroup$
    A monotonic function can only have discontinuity of the first kind
    $endgroup$
    – Li Chun Min
    May 12 '17 at 8:57










  • $begingroup$
    , which is at most countable
    $endgroup$
    – Li Chun Min
    May 12 '17 at 8:58










  • $begingroup$
    $f$ is Riemann integrable iff $f$ is continuous almost everywhere on $[0,1]$. Therefore since a monotone function is Riemann integrable 2 must be correct.
    $endgroup$
    – Janitha357
    May 12 '17 at 9:00















1












$begingroup$


Let $f:[0,1]to[0,1]$ be monotonically increasing. Which of the following statements is/are true?



  1. $f$ must be continuous at all but finitely many points in $[0,1]$

  2. $f$ must be continuous at all but countably many points in $[0,1]$

  3. $f$ must be Riemann integrable

  4. $f$ must be Lebesgue integrable

I know that set of discontinuities of monotonic function is at most countable so 1 is not true.



If the set of discontinuities is at most countable then how to conclude for the second option that is set of continuities of monotone function. Here domain is [0,1] .So the set of continuities can be uncountable?



Monotone function on $[a,b]$ is Riemann integrable and hence Lebesgue integrable hence option 3 and 4 are correct.










share|cite|improve this question











$endgroup$











  • $begingroup$
    A monotonic function can only have discontinuity of the first kind
    $endgroup$
    – Li Chun Min
    May 12 '17 at 8:57










  • $begingroup$
    , which is at most countable
    $endgroup$
    – Li Chun Min
    May 12 '17 at 8:58










  • $begingroup$
    $f$ is Riemann integrable iff $f$ is continuous almost everywhere on $[0,1]$. Therefore since a monotone function is Riemann integrable 2 must be correct.
    $endgroup$
    – Janitha357
    May 12 '17 at 9:00













1












1








1





$begingroup$


Let $f:[0,1]to[0,1]$ be monotonically increasing. Which of the following statements is/are true?



  1. $f$ must be continuous at all but finitely many points in $[0,1]$

  2. $f$ must be continuous at all but countably many points in $[0,1]$

  3. $f$ must be Riemann integrable

  4. $f$ must be Lebesgue integrable

I know that set of discontinuities of monotonic function is at most countable so 1 is not true.



If the set of discontinuities is at most countable then how to conclude for the second option that is set of continuities of monotone function. Here domain is [0,1] .So the set of continuities can be uncountable?



Monotone function on $[a,b]$ is Riemann integrable and hence Lebesgue integrable hence option 3 and 4 are correct.










share|cite|improve this question











$endgroup$




Let $f:[0,1]to[0,1]$ be monotonically increasing. Which of the following statements is/are true?



  1. $f$ must be continuous at all but finitely many points in $[0,1]$

  2. $f$ must be continuous at all but countably many points in $[0,1]$

  3. $f$ must be Riemann integrable

  4. $f$ must be Lebesgue integrable

I know that set of discontinuities of monotonic function is at most countable so 1 is not true.



If the set of discontinuities is at most countable then how to conclude for the second option that is set of continuities of monotone function. Here domain is [0,1] .So the set of continuities can be uncountable?



Monotone function on $[a,b]$ is Riemann integrable and hence Lebesgue integrable hence option 3 and 4 are correct.







monotone-functions






share|cite|improve this question















share|cite|improve this question













share|cite|improve this question




share|cite|improve this question








edited May 12 '17 at 9:08







dipali mali

















asked May 12 '17 at 8:53









dipali malidipali mali

1715




1715











  • $begingroup$
    A monotonic function can only have discontinuity of the first kind
    $endgroup$
    – Li Chun Min
    May 12 '17 at 8:57










  • $begingroup$
    , which is at most countable
    $endgroup$
    – Li Chun Min
    May 12 '17 at 8:58










  • $begingroup$
    $f$ is Riemann integrable iff $f$ is continuous almost everywhere on $[0,1]$. Therefore since a monotone function is Riemann integrable 2 must be correct.
    $endgroup$
    – Janitha357
    May 12 '17 at 9:00
















  • $begingroup$
    A monotonic function can only have discontinuity of the first kind
    $endgroup$
    – Li Chun Min
    May 12 '17 at 8:57










  • $begingroup$
    , which is at most countable
    $endgroup$
    – Li Chun Min
    May 12 '17 at 8:58










  • $begingroup$
    $f$ is Riemann integrable iff $f$ is continuous almost everywhere on $[0,1]$. Therefore since a monotone function is Riemann integrable 2 must be correct.
    $endgroup$
    – Janitha357
    May 12 '17 at 9:00















$begingroup$
A monotonic function can only have discontinuity of the first kind
$endgroup$
– Li Chun Min
May 12 '17 at 8:57




$begingroup$
A monotonic function can only have discontinuity of the first kind
$endgroup$
– Li Chun Min
May 12 '17 at 8:57












$begingroup$
, which is at most countable
$endgroup$
– Li Chun Min
May 12 '17 at 8:58




$begingroup$
, which is at most countable
$endgroup$
– Li Chun Min
May 12 '17 at 8:58












$begingroup$
$f$ is Riemann integrable iff $f$ is continuous almost everywhere on $[0,1]$. Therefore since a monotone function is Riemann integrable 2 must be correct.
$endgroup$
– Janitha357
May 12 '17 at 9:00




$begingroup$
$f$ is Riemann integrable iff $f$ is continuous almost everywhere on $[0,1]$. Therefore since a monotone function is Riemann integrable 2 must be correct.
$endgroup$
– Janitha357
May 12 '17 at 9:00










1 Answer
1






active

oldest

votes


















0












$begingroup$

Let $f(x+)=lim_yrightarrow x^+f(y)$ and $f(x-)=lim_yrightarrow x^-f(y)$. Note that $f(x+)geq f(x-)~forall x$ as $f$ is increasing. Then let $D_n=x:f(x+)-f(x-)geqfrac1n$. As $f:[0,1]rightarrow[0,1]$, hence $|D_n|leq n$. Also the set of all discontinuities, $D=cup_n=1^inftyD_n$, which is a countable union of finite sets, and hence is countable.



Also Lebesgue measure of a countable set is $0$. Hence Lebesgue measurable.






share|cite|improve this answer









$endgroup$












  • $begingroup$
    second option is correct?
    $endgroup$
    – dipali mali
    May 13 '17 at 11:17










  • $begingroup$
    yes it is correct
    $endgroup$
    – Abishanka Saha
    May 13 '17 at 11:57











Your Answer





StackExchange.ifUsing("editor", function ()
return StackExchange.using("mathjaxEditing", function ()
StackExchange.MarkdownEditor.creationCallbacks.add(function (editor, postfix)
StackExchange.mathjaxEditing.prepareWmdForMathJax(editor, postfix, [["$", "$"], ["\\(","\\)"]]);
);
);
, "mathjax-editing");

StackExchange.ready(function()
var channelOptions =
tags: "".split(" "),
id: "69"
;
initTagRenderer("".split(" "), "".split(" "), channelOptions);

StackExchange.using("externalEditor", function()
// Have to fire editor after snippets, if snippets enabled
if (StackExchange.settings.snippets.snippetsEnabled)
StackExchange.using("snippets", function()
createEditor();
);

else
createEditor();

);

function createEditor()
StackExchange.prepareEditor(
heartbeatType: 'answer',
autoActivateHeartbeat: false,
convertImagesToLinks: true,
noModals: true,
showLowRepImageUploadWarning: true,
reputationToPostImages: 10,
bindNavPrevention: true,
postfix: "",
imageUploader:
brandingHtml: "Powered by u003ca class="icon-imgur-white" href="https://imgur.com/"u003eu003c/au003e",
contentPolicyHtml: "User contributions licensed under u003ca href="https://creativecommons.org/licenses/by-sa/3.0/"u003ecc by-sa 3.0 with attribution requiredu003c/au003e u003ca href="https://stackoverflow.com/legal/content-policy"u003e(content policy)u003c/au003e",
allowUrls: true
,
noCode: true, onDemand: true,
discardSelector: ".discard-answer"
,immediatelyShowMarkdownHelp:true
);



);













draft saved

draft discarded


















StackExchange.ready(
function ()
StackExchange.openid.initPostLogin('.new-post-login', 'https%3a%2f%2fmath.stackexchange.com%2fquestions%2f2277653%2fcontinuity-of-monotonically-increasing-function%23new-answer', 'question_page');

);

Post as a guest















Required, but never shown

























1 Answer
1






active

oldest

votes








1 Answer
1






active

oldest

votes









active

oldest

votes






active

oldest

votes









0












$begingroup$

Let $f(x+)=lim_yrightarrow x^+f(y)$ and $f(x-)=lim_yrightarrow x^-f(y)$. Note that $f(x+)geq f(x-)~forall x$ as $f$ is increasing. Then let $D_n=x:f(x+)-f(x-)geqfrac1n$. As $f:[0,1]rightarrow[0,1]$, hence $|D_n|leq n$. Also the set of all discontinuities, $D=cup_n=1^inftyD_n$, which is a countable union of finite sets, and hence is countable.



Also Lebesgue measure of a countable set is $0$. Hence Lebesgue measurable.






share|cite|improve this answer









$endgroup$












  • $begingroup$
    second option is correct?
    $endgroup$
    – dipali mali
    May 13 '17 at 11:17










  • $begingroup$
    yes it is correct
    $endgroup$
    – Abishanka Saha
    May 13 '17 at 11:57















0












$begingroup$

Let $f(x+)=lim_yrightarrow x^+f(y)$ and $f(x-)=lim_yrightarrow x^-f(y)$. Note that $f(x+)geq f(x-)~forall x$ as $f$ is increasing. Then let $D_n=x:f(x+)-f(x-)geqfrac1n$. As $f:[0,1]rightarrow[0,1]$, hence $|D_n|leq n$. Also the set of all discontinuities, $D=cup_n=1^inftyD_n$, which is a countable union of finite sets, and hence is countable.



Also Lebesgue measure of a countable set is $0$. Hence Lebesgue measurable.






share|cite|improve this answer









$endgroup$












  • $begingroup$
    second option is correct?
    $endgroup$
    – dipali mali
    May 13 '17 at 11:17










  • $begingroup$
    yes it is correct
    $endgroup$
    – Abishanka Saha
    May 13 '17 at 11:57













0












0








0





$begingroup$

Let $f(x+)=lim_yrightarrow x^+f(y)$ and $f(x-)=lim_yrightarrow x^-f(y)$. Note that $f(x+)geq f(x-)~forall x$ as $f$ is increasing. Then let $D_n=x:f(x+)-f(x-)geqfrac1n$. As $f:[0,1]rightarrow[0,1]$, hence $|D_n|leq n$. Also the set of all discontinuities, $D=cup_n=1^inftyD_n$, which is a countable union of finite sets, and hence is countable.



Also Lebesgue measure of a countable set is $0$. Hence Lebesgue measurable.






share|cite|improve this answer









$endgroup$



Let $f(x+)=lim_yrightarrow x^+f(y)$ and $f(x-)=lim_yrightarrow x^-f(y)$. Note that $f(x+)geq f(x-)~forall x$ as $f$ is increasing. Then let $D_n=x:f(x+)-f(x-)geqfrac1n$. As $f:[0,1]rightarrow[0,1]$, hence $|D_n|leq n$. Also the set of all discontinuities, $D=cup_n=1^inftyD_n$, which is a countable union of finite sets, and hence is countable.



Also Lebesgue measure of a countable set is $0$. Hence Lebesgue measurable.







share|cite|improve this answer












share|cite|improve this answer



share|cite|improve this answer










answered May 12 '17 at 9:06









Abishanka SahaAbishanka Saha

7,90511022




7,90511022











  • $begingroup$
    second option is correct?
    $endgroup$
    – dipali mali
    May 13 '17 at 11:17










  • $begingroup$
    yes it is correct
    $endgroup$
    – Abishanka Saha
    May 13 '17 at 11:57
















  • $begingroup$
    second option is correct?
    $endgroup$
    – dipali mali
    May 13 '17 at 11:17










  • $begingroup$
    yes it is correct
    $endgroup$
    – Abishanka Saha
    May 13 '17 at 11:57















$begingroup$
second option is correct?
$endgroup$
– dipali mali
May 13 '17 at 11:17




$begingroup$
second option is correct?
$endgroup$
– dipali mali
May 13 '17 at 11:17












$begingroup$
yes it is correct
$endgroup$
– Abishanka Saha
May 13 '17 at 11:57




$begingroup$
yes it is correct
$endgroup$
– Abishanka Saha
May 13 '17 at 11:57

















draft saved

draft discarded
















































Thanks for contributing an answer to Mathematics Stack Exchange!


  • Please be sure to answer the question. Provide details and share your research!

But avoid


  • Asking for help, clarification, or responding to other answers.

  • Making statements based on opinion; back them up with references or personal experience.

Use MathJax to format equations. MathJax reference.


To learn more, see our tips on writing great answers.




draft saved


draft discarded














StackExchange.ready(
function ()
StackExchange.openid.initPostLogin('.new-post-login', 'https%3a%2f%2fmath.stackexchange.com%2fquestions%2f2277653%2fcontinuity-of-monotonically-increasing-function%23new-answer', 'question_page');

);

Post as a guest















Required, but never shown





















































Required, but never shown














Required, but never shown












Required, but never shown







Required, but never shown

































Required, but never shown














Required, but never shown












Required, but never shown







Required, but never shown







Popular posts from this blog

Lowndes Grove History Architecture References Navigation menu32°48′6″N 79°57′58″W / 32.80167°N 79.96611°W / 32.80167; -79.9661132°48′6″N 79°57′58″W / 32.80167°N 79.96611°W / 32.80167; -79.9661178002500"National Register Information System"Historic houses of South Carolina"Lowndes Grove""+32° 48' 6.00", −79° 57' 58.00""Lowndes Grove, Charleston County (260 St. Margaret St., Charleston)""Lowndes Grove"The Charleston ExpositionIt Happened in South Carolina"Lowndes Grove (House), Saint Margaret Street & Sixth Avenue, Charleston, Charleston County, SC(Photographs)"Plantations of the Carolina Low Countrye

random experiment with two different functions on unit interval Announcing the arrival of Valued Associate #679: Cesar Manara Planned maintenance scheduled April 23, 2019 at 00:00UTC (8:00pm US/Eastern)Random variable and probability space notionsRandom Walk with EdgesFinding functions where the increase over a random interval is Poisson distributedNumber of days until dayCan an observed event in fact be of zero probability?Unit random processmodels of coins and uniform distributionHow to get the number of successes given $n$ trials , probability $P$ and a random variable $X$Absorbing Markov chain in a computer. Is “almost every” turned into always convergence in computer executions?Stopped random walk is not uniformly integrable

How should I support this large drywall patch? Planned maintenance scheduled April 23, 2019 at 00:00UTC (8:00pm US/Eastern) Announcing the arrival of Valued Associate #679: Cesar Manara Unicorn Meta Zoo #1: Why another podcast?How do I cover large gaps in drywall?How do I keep drywall around a patch from crumbling?Can I glue a second layer of drywall?How to patch long strip on drywall?Large drywall patch: how to avoid bulging seams?Drywall Mesh Patch vs. Bulge? To remove or not to remove?How to fix this drywall job?Prep drywall before backsplashWhat's the best way to fix this horrible drywall patch job?Drywall patching using 3M Patch Plus Primer